K
Khách

Hãy nhập câu hỏi của bạn vào đây, nếu là tài khoản VIP, bạn sẽ được ưu tiên trả lời.

16 tháng 10 2020

Xí trước phần b

Ta có: \(\frac{1}{a^3\left(b+c\right)}+\frac{1}{b^3\left(c+a\right)}+\frac{1}{c^3\left(a+b\right)}\)

\(=\frac{abc}{a^3\left(b+c\right)}+\frac{abc}{b^3\left(c+a\right)}+\frac{abc}{c^3\left(a+b\right)}\)

\(=\frac{bc}{a^2b+ca^2}+\frac{ca}{b^2c+ab^2}+\frac{ab}{c^2a+bc^2}\)

\(=\frac{b^2c^2}{a^2b^2c+a^2bc^2}+\frac{c^2a^2}{ab^2c^2+a^2b^2c}+\frac{a^2b^2}{a^2bc^2+ab^2c^2}\)

\(=\frac{\left(bc\right)^2}{ab+ca}+\frac{\left(ca\right)^2}{bc+ab}+\frac{\left(ab\right)^2}{ca+bc}\)

\(\ge\frac{\left(bc+ca+ab\right)^2}{2\left(ab+bc+ca\right)}=\frac{ab+bc+ca}{2}\ge\frac{3\sqrt[3]{\left(abc\right)^2}}{2}=\frac{3}{2}\)

Dấu "=" xảy ra khi: \(a=b=c=1\)

16 tháng 10 2020

Cách làm khác của phần b ngắn gọn hơn:)

Ta có; \(\frac{1}{a^3\left(b+c\right)}+\frac{1}{b^3\left(c+a\right)}+\frac{1}{c^3\left(a+b\right)}\)

\(=\frac{\frac{1}{a^2}}{a\left(b+c\right)}+\frac{\frac{1}{b^2}}{b\left(c+a\right)}+\frac{\frac{1}{c^2}}{c\left(a+b\right)}\)

\(=\frac{\left(\frac{1}{a}\right)^2}{ab+ca}+\frac{\left(\frac{1}{b}\right)^2}{bc+ab}+\frac{\left(\frac{1}{c}\right)^2}{ca+bc}\)

\(\ge\frac{\left(\frac{1}{a}+\frac{1}{b}+\frac{1}{c}\right)^2}{2\left(ab+bc+ca\right)}=\frac{\left(\frac{ab+bc+ca}{abc}\right)^2}{2\left(ab+bc+ca\right)}=\frac{ab+bc+ca}{2}\ge\frac{3\sqrt[3]{\left(abc\right)^2}}{2}=\frac{3}{2}\)

Dấu "=" xảy ra khi: a = b = c = 1

18 tháng 3 2020

Áp dụng BĐT Svacxo ta có :

\(\frac{1}{a^3\left(7b+3c\right)}+\frac{1}{b^3\left(7c+3a\right)}+\frac{1}{c^3\left(7a+3b\right)}=\frac{\frac{1}{a^2}}{7ab+7ac}+\frac{\frac{1}{b^2}}{7bc+3ab}+\frac{\frac{1}{c^2}}{7ac+3bc}\)

\(\ge\frac{\left(\frac{1}{a}+\frac{1}{b}+\frac{1}{c}\right)^2}{10\left(ab+bc+ca\right)}=\frac{1}{10}.\frac{\left(\frac{1}{a}+\frac{1}{b}+\frac{1}{c}\right)^2}{ab+bc+ca}=\frac{1}{10}.\left(ab+bc+ca\right)\)

\(=\frac{1}{10}.\frac{ab+bc+ca}{abc}=\frac{1}{10}.\left(\frac{1}{a}+\frac{1}{b}+\frac{1}{c}\right)\left(đpcm\right)\)

Dấu " = " xảy ra \(\Leftrightarrow a=b=c=1\)

8 tháng 1 2020

\(\left(a+b\right)\left(b+c\right)\left(c+a\right)+abc\)

\(=abc+a^2b+ab^2+a^2c+ac^2+b^2c+bc^2+abc+abc\)

\(=\left(a+b+c\right)\left(ab+bc+ca\right)\)( phân tích nhân tử các kiểu )

\(\Rightarrow\left(a+b\right)\left(b+c\right)\left(c+a\right)\ge\left(a+b+c\right)\left(ab+bc+ca\right)-abc\left(1\right)\)

\(a+b+c\ge3\sqrt[3]{abc};ab+bc+ca\ge3\sqrt[3]{a^2b^2c^2}\Rightarrow\left(a+b+c\right)\left(ab+bc+ca\right)\ge9abc\)

\(\Rightarrow-abc\ge\frac{-\left(a+b+c\right)\left(ab+bc+ca\right)}{9}\)

Khi đó:\(\left(a+b+c\right)\left(ab+bc+ca\right)-abc\)

\(\ge\left(a+b+c\right)\left(ab+bc+ca\right)-\frac{\left(a+b+c\right)\left(ab+bc+ca\right)}{9}\)

\(=\frac{8\left(a+b+c\right)\left(ab+bc+ca\right)}{9}\left(2\right)\)

Từ ( 1 ) và ( 2 ) có đpcm

25 tháng 3 2018

BĐT\(\Leftrightarrow\frac{abc}{a^3\left(b+c\right)}+\frac{abc}{b^3\left(a+c\right)}+\frac{abc}{c^3\left(a+b\right)}\ge\frac{1}{2}\left(\frac{1}{a}+\frac{1}{b}+\frac{1}{c}\right)\)

\(\Leftrightarrow\frac{\frac{1}{a^2}+\frac{1}{b^2}+\frac{1}{c^2}}{\frac{1}{b}+\frac{1}{c}.\frac{1}{a}+\frac{1}{c}.\frac{1}{a}+\frac{1}{b}}\ge\frac{1}{2}\left(\frac{1}{a}+\frac{1}{b}+\frac{1}{c}\right)\)

Đặt \(x=\frac{1}{a};y=\frac{1}{b};z=\frac{1}{c}\). Áp dụng BĐT: AM-GM ta có:

\(\frac{a^2}{b+c}+\frac{b+c}{4}\ge2\sqrt{\frac{a^2}{b+c}.\frac{b+c}{4}}=a\)

\(\frac{b^2}{a+b}+\frac{a+c}{4}\ge2\sqrt{\frac{b^2}{a+b}.\frac{a+b}{4}}=b\)

\(\frac{c^2}{a+b}+\frac{a+b}{4}\ge2\sqrt{\frac{c^2}{a+b}+\frac{a+b}{4}}=c\)

Cộng theo vế 3 BĐT trên ta có:

\(\frac{a^2}{b+c}+\frac{b^2}{a+c}+\frac{c^2}{a+b}\ge\frac{1}{a}+\frac{1}{b}+\frac{1}{c}\)

hay \(\frac{a^2}{b+c}+\frac{b^2}{a+c}+\frac{c^2}{a+b}\ge\frac{3}{2}\)

Dấu bằng = xảy ra khi a = b = c = 1

5 tháng 8 2020

Đặt  \(x=\frac{1}{a};y=\frac{1}{b};z=\frac{1}{c}\Rightarrow xyz=1;x>0;y>0;z>0\)

Ta cần chứng minh bất đẳng thức sau : \(A=\frac{x^2}{y+z}+\frac{y^2}{x+z}+\frac{z^2}{x+y}\ge\frac{3}{2}\)

Sử dụng bất đẳng thức Bunhiacopxki cho 2 bộ số :

\(\left(\sqrt{y+z};\sqrt{z+x};\sqrt{x+y}\right);\left(\frac{x}{\sqrt{y+z}};\frac{y}{\sqrt{z+x}};\frac{z}{\sqrt{x+y}}\right)\)

Ta có : \(\left(x+y+z\right)^2\le\left(x+y+z+x+y+z\right)A\)

\(\Rightarrow A\ge\frac{x+y+z}{2}\ge\frac{3\sqrt[3]{xyz}}{2}=\frac{3}{2}\left(Q.E.D\right)\)

Đẳng thức xảy ra khi và chỉ khi \(x=y=z=1\Leftrightarrow a=b=c=1\)

27 tháng 8 2019

1/ Đặt

\(\frac{a}{b^2}=x,\frac{b}{c^2}=y,\frac{c}{a^2}=z,xyz=1\)thì ta có

\(x+y+z=\frac{1}{x}+\frac{1}{y}+\frac{1}{z}\)

\(\Leftrightarrow xy+yz+zx=x+y+z\)

\(\Leftrightarrow xyz-xy-yz-zx+x+y+z-1=0\)

\(\Leftrightarrow\left(x-1\right)\left(y-1\right)\left(z-1\right)=0\)

\(\Leftrightarrow x=1;y=1;z=1\)

\(\Rightarrow\frac{a}{b^2}=1;\frac{b}{c^2}=1;\frac{c}{a^2}=1\)

\(\Leftrightarrow a=b^2;b=c^2;c=a^2\)

27 tháng 8 2019

2/ Đặt

\(ab=x,bc=y,ca=z\) cần tính

\(P=\left(1+\frac{z}{y}\right)\left(1+\frac{x}{z}\right)\left(1+\frac{y}{x}\right)\)

\(\Rightarrow x^3+y^3+z^3=3xyz\)

\(\Leftrightarrow\left(x+y+z\right)\left(x^2+y^2+z^2-xy-yz-zx\right)=0\)

\(\Leftrightarrow\orbr{\begin{cases}x+y+z=0\\x^2+y^2+z^2-xy-yz-zx=0\end{cases}}\)

Xét \(x+y+z=0\)

\(\Rightarrow P=\frac{x+y}{x}.\frac{y+z}{y}.\frac{z+x}{z}=\frac{\left(-x\right)\left(-y\right)\left(-z\right)}{xyz}=-1\)

Xét \(x^2+y^2+z^2-xy-yz-zx=0\)

\(\Leftrightarrow2\left(x^2+y^2+z^2-xy-yz-zx\right)=0\)

\(\Leftrightarrow\left(x-y\right)^2+\left(y-z\right)^2+\left(z-x\right)^2=0\)

\(\Leftrightarrow x=y=z\)

\(\Rightarrow P=\left(1+1\right)\left(1+1\right)\left(1+1\right)=8\)

24 tháng 10 2020

Giúp mình với các bạn ơiii

24 tháng 10 2020

Theo bất đẳng thức AM - GM, ta có: \(\frac{a^3}{\left(1+b\right)\left(1+c\right)}+\frac{1+b}{8}+\frac{1+c}{8}\ge3\sqrt[3]{\frac{a^3}{\left(1+b\right)\left(1+c\right)}.\frac{1+b}{8}.\frac{1+c}{8}}=\frac{3}{4}a\Rightarrow\frac{a^3}{\left(1+b\right)\left(1+c\right)}\ge\frac{3a}{4}-\frac{b+c}{8}-\frac{1}{4}\)Tương tự, ta được: \(\frac{b^3}{\left(1+c\right)\left(1+a\right)}\ge\frac{3b}{4}-\frac{c+a}{8}-\frac{1}{4}\)\(\frac{c^3}{\left(1+a\right)\left(1+b\right)}\ge\frac{3c}{4}-\frac{a+b}{8}-\frac{1}{4}\)

Cộng vế theo vế ba bất đẳng thức trên, ta được: \(\frac{a^3}{\left(1+b\right)\left(1+c\right)}+\frac{b^3}{\left(1+c\right)\left(1+a\right)}+\frac{c^3}{\left(1+a\right)\left(1+b\right)}\ge\frac{3}{4}\left(a+b+c\right)-\frac{1}{4}\left(a+b+c\right)-\frac{3}{4}=\frac{1}{2}\left(a+b+c\right)-\frac{3}{4}\ge\frac{1}{2}.3\sqrt[3]{abc}-\frac{3}{4}=\frac{3}{4}\)Đẳng thức xảy ra khi a = b = c = 1